Find the area to the left of z = 0.25.
A. 0.6012 B. 0.5987 C. 0.4013 D.0.3988

Answers

Answer 1

Answer:

.5987

Step-by-step explanation:

Use a ztable and find .25 (pic below)


Related Questions

two triangles are similar what is x

Answers

Answer:

x = 10

Step-by-step explanation:

smaller triangle / bigger triangle = 20 / 28

hence,

3x / (4x+2) = 20/28

28(3x) = 20(4x+2)

84x = 80x + 40

4x = 40

x = 10

y= -(x+3)^2 -5
What is the leading coefficient?
How do you find the vertex?

Answers

Answer:

To find the leading coefficient, first expand the function:

[tex]y= -(x+3)^{2} -5\\\\y=-(x^{2} +6x+9)-5\\\\y=-x^{2} -6x-9-5\\\\y=-x^{2} -6x-14[/tex]

The leading coefficient is the coefficient of the highest-order term, which, in this case, would be the -1 from -x².

To find the vertex: see image below

Vertex = (-3, -5)

3. If triangle ABC has the following measurements, find the measure of angle A.
a = 17
b = 21
C = 25

Answers

9514 1404 393

Answer:

  (a)  42.3°

Step-by-step explanation:

Side 'a' is the shortest of three unequal sides, so angle A will be the smallest angle in the triangle. Its measure can be found from the Law of Cosines.

  a² = b² +c² -2bc·cos(A)

  cos(A) = (b² +c² -a²)/(2bc) = (21² +25² -17²)/(2·21·25) = 777/1050

  A = arccos(777/1050) ≈ 42.3°

The measure of angle A is about 42.3°.

_____

Additional comment

The smallest angle in a triangle can never be greater than 60°. This lets you eliminate choices that exceed that value.

Answer:

 (a)  42.3°

Step-by-step explanation:

Find the circumference and the area of a circle with diameter equal to 8.6 inches. Use 3.14 for pi

Please answer it will mean a lot thanks

Answers

Answer: Circumference of circle = 27.004 inches

              Area of circle = 58.0586 inches²

Step-by-step explanation:

Diameter of circle = 8.6 inches

Pi ([tex]\pi[/tex]) = 3.14

Circumference of circle (With diameter) = [tex]\pi \\[/tex]d ([tex]\pi[/tex]×diameter)

                                                                = 3.14 × 8.6

                                                                = 27.004 inches

Area of circle (With diameter) = [tex]\pi[/tex][tex]d^{2}[/tex]/4

                                                 = 3.14 × 8.6 × 8.6 / 4

                                                 = 3.14 × 73.96 / 4

                                                 = 58.0586 inches²

URGENT ! HELP ME I WILL MARK YOU BRAINLIEST !!!!

pleasee fasterrr !!!!​

Answers

Answer:

3b (3a - 4b)

Step-by-step explanation:

Answer:

C

Step-by-step explanation:

9ab - 12b² ← factor out 3b from each term

= 3b(3a - 4b) → C

Help solve for the area

Answers

Answer:

B

Step-by-step explanation:

half × base × height

height × length

Answer: B

Step-by-step explanation:

Triangle)

25 - 7 = 18

[tex]A=\frac{1}{2}(b)(h)\\A=\frac{1}{2}(18)(17)\\A=153cm^2[/tex]

Rectangle)

[tex]A=b(h)\\A=7(17) = 119cm^2[/tex]

Total)

[tex]153+119=272 cm^2[/tex]

Ms. Dawson’s call did a science experiment. The class started out with 650 bacteria cells. The growth rate predicted was 4.5%. Sketch the graph that represents the situation. Label the y-intercept and the point that represents the projected bacteria population 30 h from the start of the experiment. Round to the nearest whole number.

Answers

Answer:

your slope would be 4.5.... so go up 4 and to the right 5. the y-intercept is 650 so that is where your line would start instead of at 0... hope this helped :)

Step-by-step explanation:

The exponential function gotten from the table is given by y = 650(1.045)ˣ

Exponential function

An exponential function is in the form:

y = abˣ

where y, x are variables, a is the initial value of y and b is the multipliers.

Let y represent the bacteria population after x hours.

The class started out with 650 bacteria cells.

a = 650

Growth rate = 4.5%

b = 100% + 4.5% = 104.5% = 1.045

The exponential function gotten from the table is given by y = 650(1.045)ˣ

After 30 hours:

y = 650(1.045)³⁰ = 2434

Find out more on exponential function at: https://brainly.com/question/12940982

find the value of x. give reasons to justify your answer NEED HELP ASAP!!!!

Answers

Answer:

[tex]x = 34^\circ[/tex]

Step-by-step explanation:

Note that ∠TSU and ∠PSR are vertical angles. Hence:

[tex]m\angle TSU = m\angle PSR[/tex]

∠PSR is the sum of ∠PSQ and ∠QSR. Hence:

[tex]\displaystyle m\angle TSU = m\angle PSQ + m\angle QSR[/tex]

We know that ∠TSU measures 4x and ∠QSR measures 3x. Thus:

[tex](4x) = m\angle PSQ + (3x)[/tex]

Solve for ∠PSQ:

[tex]m\angle PSQ = x[/tex]

Next, ∠PQS and ∠RQS form a linear pair. Thus:

[tex]m\angle PQS + m\angle RQS = 180^\circ[/tex]

∠RQS measures 68°. Thus:

[tex]m\angle PQS +(68^\circ) = 180^\circ[/tex]

Solve for ∠PQS:

[tex]m\angle PQS = 112^\circ[/tex]

The interior angles of a triangle must total 180°. So, for ΔPQS:

[tex]\displaystyle m\angle SPQ + m\angle PQS + m\angle PSQ = 180^\circ[/tex]

Substitute in the known values:

[tex](x) + (112^\circ) + (x) = 180^\circ[/tex]

Simplify:

[tex]2x = 68^\circ[/tex]

And divide. Hence:

[tex]x = 34^\circ[/tex]

(f^3-5f+25)-(4f^2-12f+9)

Answers

Answer:

3−42+7+16

Step-by-step explanation:

if its simplify

Which graph represents the function f(x)=|x−1|−3 ?

Answers

Answer is B, (the one on the top right)

In the equation $\frac{1}{j} + \frac{1}{k} = \frac{1}{3}$, both $j$ and $k$ are positive integers. What is the sum of all possible values for $k$?

Answers

Answer:

Hello,

answer 22

Step-by-step explanation:

[tex]\dfrac{1}{j} +\dfrac{1}{k} =\dfrac{1}{3} \\\\\dfrac{k+j}{j*k} =\dfrac{1}{3} \\\\\\3k+3j=j*k\\\\k(3-j)=-3j\\\\k=\dfrac{3j}{j-3} \\\\k=\dfrac{3j-9+9}{j-3} \\\\k=3+\dfrac{9}{j-3} \\\\\\j-3\ must\ be \ a divisor\ of\ 9 ==> 1,3,9\\\\j-3=1 ==> j=4 , k=3+\dfrac{9}{1} =12\\\\j-3=3 ==> j=6 , k=3+\dfrac{9}{3} =6\\\\j-3=9 ==> j=12 , k=3+\dfrac{9}{9} =4\\\\\sum\ k=12+6+4=22\\[/tex]

The sum of all possible values for k is 22.

What is fraction?

The fractional bar is a horizontal bar that divides the numerator and denominator of every fraction into these two halves.

The number of parts into which the whole has been divided is shown by the denominator. It is positioned in the fraction's lower portion, below the fractional bar.How many sections of the fraction are displayed or chosen is shown in the numerator. It is positioned above the fractional bar in the upper portion of the fraction.

Given:

1/j + 1/k= 1/3

Simplifying the fraction

(k+ j)/ kj= 1/3

3k + 3j = kj

k(3- j) = -3j

k = -3j/ (3- j)

k= 3j/ (j-3)

k = 3j + 9 - 9 / (j-3)

k = 3 + 9/ (j-3)

Since (j-3) must be divisor of 9.

j- 3 = 1---> j=4, k= 3 +9 = 12

j- 3 = 3---> j=6, k= 3 +9/3 = 6

j- 3 = 9---> j=12, k= 3 +9/9 = 4

So, The sum is = 12+ 6 + 4 = 22

Learn more about Fraction here:

https://brainly.com/question/10354322

#SPJ6

pls help me with this question and reply answer​

Answers

Step-by-step explanation:

From both the cases we can say that √n is either a natural number or an irrational number. Hence the correct option is (d)Either a natural number or an irrational number. Note: Before solving this type of question we should have proper knowledge of natural numbers, rational numbers and irrational numbers.

The width of a rectangle is 9
inches less than the length. The
perimeter is 86 inches

Answers

Answer:

Below.

Step-by-step explanation:

If the length is x then the width is x-9 inches.

Perimeter

= 2L + 2W = 86

2x + 2(x - 9) = 86

4x - 18 = 86

4x = 104

x = 26

So the length is 26 inches and the width is 17 inches.

what is value of y if 2x+3y=4​

Answers

Answer:

y=(4-2x)/3

Step-by-step explanation:

3y= 4-2x

y= (4-2x)/3

Jo bought a used car for $6000 and paid a 15% deposit. How much did he still have to pay?

Answers

Answer:

900 is the correct awnser

John and Pablo caught fish that have the lengths, in centimeters, listed below. 45, 44, 47, 49, 45, 47, 42, 39, 45, 42, 44 Which box-and-whisker plot correctly represents the data?

Answers

The options for the box and whisker plots aren't given ; however using technology, a box and whisker plot could be generated from the data.

Answer:

Step-by-step explanation:

Given :

45, 44, 47, 49, 45, 47, 42, 39, 45, 42, 44

Using technology, the box and whisker plot generated for the data is attached below.

The 5 - number summary is also given below :

Minimum: 39

Median: 45

First quartile: 42

Third quartile: 47

Interquartile Range: 5

Maximum: 49

Outliers: none

Answer:

Step-by-step explanation:

[tex](a+b)^{2}[/tex]

Answers

Answer:

[tex] ({a + b})^{2} [/tex]

[tex](a + b)(a + b)[/tex]

[tex] {a}^{2} + 2ab + {b}^{2} [/tex]

hope this help you

State the slope of the line shown below (help ASAP)
Is it a, b, c or d?

Answers

Answer:

D

Step-by-step explanation:

(the above graph depicts a horizontal line that intersects the y axis at -2)

(so, y = -2)

Answer:

0

Step-by-step explanation:

Horizontal line x axis is 0 while the vertical y axis is undefined.

Find the length of the third side. If necessary, round to the nearest tenth. 5 10 ​

Answers

Answer:

[tex]\boxed {\boxed {\sf 8.7}}[/tex]

Step-by-step explanation:

We are asked to find the length of the third side in a triangle, given the other 2 sides.

Since this is a right triangle (note the small square in the corner of the triangle representing a 90 degree /right angle), we can use the Pythagorean Theorem.

[tex]a^2 + b^2 =c^2[/tex]

In this theorem, a and b are the legs of the triangle and c is the hypotenuse.

We know that the unknown side (we can say it is a) and the side measuring 5 are the legs because they form the right angle. The side measuring 10 is the hypotenuse because it is opposite the right angle.

b= 5 c= 10

Substitute the values into the formula.

[tex]a^2 + (5)^2 = (10)^2[/tex]

Solve the exponents.

(5)²= 5*5 = 25 (10)²= 10*10= 100

[tex]a^2 + 25=100[/tex]

We are solving for a, so we must isolate the variable. 25 is being added to a. The inverse operation of addition is subtraction, so we subtract 25 from both sides.

[tex]a^2 +25-25=100-25[/tex]

[tex]a^2=100-25[/tex]

[tex]a^2 = 75[/tex]

a is being squared. The inverse of a square is the square root, so we take the square root of both sides.

[tex]\sqrt {a^2}= \sqrt{75}[/tex]

[tex]a= \sqrt{75}[/tex]

[tex]a= 8.660254038[/tex]

Round to the nearest tenth. The 6 in the hundredth place tells us to round the 6 up to a 7 in the tenth place.

[tex]a \approx 8.7[/tex]

The length of the third side is approximately 8.7

Perpendicular=P=5Hypontenuse=H=10Base=B=?

Using Pythagorean theorem

[tex]\boxed{\sf B^2=H^2-P^2}[/tex]

Putting values

[tex]\\ \sf \longmapsto B^2=10^2-5^2[/tex]

[tex]\\ \sf \longmapsto B^2=100-25[/tex]

[tex]\\ \sf \longmapsto B^2=75[/tex]

[tex]\\ \sf \longmapsto B=\sqrt{75}[/tex]

[tex]\\ \sf \longmapsto B=\sqrt{25\times 3}[/tex]

[tex]\\ \sf \longmapsto B=5\sqrt{3}[/tex]

[tex]\\ \sf \longmapsto B=5\times 1.732[/tex]

[tex]\\ \sf \longmapsto B=8.66[/tex]

[tex]\\ \sf \longmapsto B\approx 8.7[/tex]

What’s this topic called

Answers

Answer: MATH

Step-by-step explanation:

Answer:

answer for the question is

x = 1

y = 4

subject is math btw if you are asking about that

What is an
equation of the line that passes through the points (-3,-1) and (-4,-4)

Answers

Answer:

y= 3x+8

Step-by-step explanation:

not a 100% sure...

sry if its wrong

(try using Math-way, its rly helpful)

Answer:

Step-by-step explanation:

y=mx+b

To find slope: -4+1/-4+3

Slope=3

y=3x+b

Plug in either points ,as an example, i'll plug in (-3,-1)

-1=3(-3)+b

-1=-9+b

8=b

Finished formula: y=3x+8

Priya bought a football for £3.50.
She received £1.50 change.
How much money did she give the shop assistant?
£10.00
£4.00
£5.00
Which one £10.00, £4.00 or £5.00?

Answers

Answer:

$5.00

Step-by-step explanation:

$3.50+$1.50=$5.00

She gave them £5.00
£5 - £1.50 = £3.50

The ratio of girls to boys in grade 6 was 3:2 at Lincoln middle school last year. There were 60 kids in grade 6. How many in grade 6 were girls

Answers

Answer:

40 girls 20 boys i think / guess

Step-by-step explanation:

Answer:

20/40

Step-by-step explanation:

i hope it help

Please Help

Select the correct answer.

What is the slope of a line parallel to line r?

A. A line parallel to line r has an undefined slope.
B. 0
C. -1
D. 1

Answers

Answer:

B 0

Step-by-step explanation:

r is a horizontal line

horizontal lines are of the form y= constant

The slope for horizontal lines are 0

Parallel lines have the same slope.

Answer:

B 0

Step-by-step explanation:

the slope of a line is the rise divided by the run, or the Y increase divided by the X increase. the fact that the Y value does not increase makes the slope 0

Please solve fast. It's really easy!

Answers

Answer:

j = 10.1

Step-by-step explanation:

The difference between 9.5 and 11.3 is

11.3 - 9.5 = 1.8

This is split into 3 divisions, that is

1.8 ÷ 3 = 0.6

Then

j = 9.5 + 0.6 = 10.1

Write an equation of the line that is parallel to the given line and passes through the given point.

9. y= -2x+6 (0,-4)

10. -2x+3y=12 (3,2)


pls help me!!!!!

Answers

Answer:

9.) y=-2x-4

10.)y=2/3x

Step-by-step explanation:

Parallel lines have the same slope, different intercepts

9.) The slope is -2x

To find the parallel line

y-(-4)=-2(x-0)

y+4=-2x

y-4=-2x-4

y=-2x-4

10.) You must first put the equation in slope-intercept form.

-2x+2x+3y=2x+12

3y=2x+12

Divide by 3

3y/3=2x/3+12/3

Simplify

y=2/3x+4

Slope = 2/3x

y-2=2/3(x-3)

y-2=2/3x-6/3

y-2=2/3x-2

y-2+2=2/3x-2+2

y=2/3x

find the shaded area ..

Answers

Answer:

39°

Step-by-step explanation:

The angle adjacent to 115° in the lower triangle = 180° - 115° = 65°

angle on circle in lower triangle

= 180° - (65 + 76)°  ← angle sum in triangle

= 180° - 141° = 39°

Angles on the circle subtended from the same arc are congruent, then

shaded angle = 39°

x^{4} - 5x^(2) + 2x + 3 < 0

Answers

the Answer is irrational number

Graph the system of inequalities. Then state whether the situation is infeasible, has alternate optimal solutions, or is unbounded. (Assume that x>0 y>0

Answers

Answer:

x

[tex]x \leqslant - y + 1[/tex]

Steps are show in the picture above.

Once upon a time, there was a former whose family raised pigs and chickens. One morning, the farmer's wife looked into the barnyard and told her husband that she counted 24 heads and 80 feet. How many chickens are in the yard? How many pigs are in the yard?

Answers

Using a system of equations, it is found that:

There are 8 chickens in the yard.There are 16 pigs in the yard.

-----------------------------------------

This question is solved using a system of equations. I am going to say that:

x is the number of chickens.y is the number of pigs.

-----------------------------------------

A chicken has 1 head, as do a pig.There are 24 heads.

Thus:

[tex]x + y = 24 \rightarrow y = 24 - x[/tex]

-----------------------------------------

A chicken has 2 feet.A pig has 4 feet.In total, there are 80 feet.

Thus:

[tex]2x + 4y = 80[/tex]

-----------------------------------------

Number of chicken:

Since [tex]y = 24 - x[/tex]

[tex]2x + 4y = 80[/tex]

[tex]2x + 4(24 - x) = 80[/tex]

[tex]2x + 96 - 4x = 80[/tex]

[tex]2x = 16[/tex]

[tex]x = \frac{16}{2}[/tex]

[tex]x = 8[/tex]

There are 8 chickens in the yard.

-----------------------------------------

Number of pigs:

[tex]y = 24 - x = 24 - 8 = 16[/tex]

There are 16 pigs in the yard.

A similar problem is given at https://brainly.com/question/24104709

Other Questions
What is a second-generation Japanese American called?isseiniseidesuossuTHE ANSWER IS B) NIESEI I JUST DIDNT SEE IT ON HERE SO IM ADDING IT HELP PLEASE!!! The expected value of a random variable X is 35. The variable is transformedby multiplying X by 4 and then adding 1 to it. Find the expected value (mean)of the transformed variable. A. 135 B.117 C. 154 D.141 which transformation of the red triangle on the graph maps it into the missing peice of the square?A. a translation 16 units right B. a reflection across the y-axisC. a 90 counterclockwise rotation about the origin D. a 90 clockwise rotation about the origin E. a 180 rotation about the origin Do leaves have other than. Green colour also have chlorophyll HELP ME PLZ Which of the following describes the structure of a fish's swim bladder? A. Expands and contracts to allow the fish to rise or sink in the water B. Internal organ filled with liquid c. Holds urine until it can be released into the environment D. Internal organ filled with air sacs In literature The sequence of event in a play is called [tex]3f^{2} - 15f - 108[/tex] It is well-known that retaining an existing customer is far less expensive than acquiring a new one, so you suggest that the owners begin by calculating share of customer to identify opportunities to sell more products to existing customers. Before you can calculate Camp Plus's share of customer, you need to determine what type of data you need to gather. What data sources will you need to utilize to calculate share of customer What are the steps to this problem (along with the answer)? Prelab InformationPurpose Explore the molecular process of building proteins from the information carried by RNAusing a laboratory procedure.Time Approximately 45 minutesQuestion How are proteins built using the information provided by a molecule of RNA?Prediction RNA determines the sequence of amino acids in proteins and polypeptides by a two-step process: transcription of DNA produces mRNA in the nucleus, then translationof the mRNA to tRNA takes place at the ribosome in the cytoplasm. 4. PLEASE HELP MEWhich of the quadratic functions has the widest graph?A. y= -4/5x2B. y= -4x2C. y= 1/3x2D. y= 0.3x2 Can someone please help me out Triangles P Q R and S T U are shown. Angles P R Q and T S U are right angles. The length of P Q is 20, the length of Q R is 16, and the length of P R is 12. The length of S T is 30, the length of T U is 34, and the length of S U is 16.Using the side lengths of PQR and STU, which angle has a sine ratio of Four-fifths?PQTU un litro de un gas es calentado a presin constante desde 20C hasta 60C que volumen final ocupar dicho gas? Which of these sentences uses commas correctly?A.Mary asked "What time do we need to be there?" B.Mary asked, what time do we need to be there? C.Mary asked what time do we need to be there? D.Mary asked, "What time do we need to be there?" convert 2m 50cm 15mm in cm You wait in line for hours to get the new special edition Nikes for $250, but you have to pay 5.3% in Virginia state sales tax. What is the total you will pay? If (0, pi/2) and tan(pi cos) = cot(pi sin), then cos(- pi/4) is equal to?\ (c+d)^2+11(c+d)+30Factor completely. 1. my / at / aunt / half / was / past / at / nine / home__________________________________________________2. a / her / quarter / parents / to / were / seven / at__________________________________________________3. sister / your / yesterday / was / evening / where / ?__________________________________________________4. the / our / not / in / children / were / library __________________________________________________5. outside / his / last / cousins / night / were / ?